Another method of saving time on simple questions – use data given in one statement to examine the other!
Now you might think we have lost it! After all, you know very well that in Data Sufficiency questions of GMAT, you must examine each statement independently. You CANNOT use data from one to analyze the other – absolutely correct. So you should ignore the other statement completely while examining one – hmm, not necessarily!
Sometimes, one statement could give us ideas about the next one such that we could save time while examining it. Needless to say, we need to be very careful but it certainly is a useful strategy. Also, it could help us verify that our calculations are correct. Here is why…
When we say DS question, think of a puzzle. The question stem gives you the statement of a puzzle ending with something like “What is the value of ?” or “Is 7?” etc. You have to answer the question asked in the puzzle. Think of the two statements that come with the question as clues to the puzzle. So the puzzlemaster gives you the first clue (statement 1) and asks you: can you answer the question now? If you are able to, your answer is either (A) or (D).
Then he tells you to ignore the first clue and gives you another clue (statement 2). Again he asks you: can you answer the question now? Again, you may or may not able to. If you are able to, your answer will be (B) or (D) depending on how you fared in statement 1. If you are unable to answer the question, he tells you to consider both statements together and then try to answer. If you are able to, your answer is (C).
The point to note here is that both clues lead you to answer the same puzzle. Say if the puzzle is: What is ? If clue 1 tells you that is 6, clue 2 cannot tell you that is 9. They both must lead you to the same value of . Clue 1 could tell you that is either 6 or 8 and clue 2 could tell you that is either 8 or 9. In this case, when we use both clues together, we find that must be 8 to satisfy both. Hence the statements never contradict each other. This means, if we get possible values of from statement 1, we know that statement 2 will also give us at least one of those values.
This is how one statement could give us a starting point for the next one. Now that you understand the “why”, let’s go on to “how”, using a question.
Question: If is a positive integer less than 10 and , what is the value of ?
Statement 1: is divisible by 3
Statement 2: is divisible by 7
Solution:
Given:
So could range from 4322 (when ) to 4330 (when ). To find the value of , we need to find the unique value of .
Statement 1 tells us that is divisible by 3.
4321 is not divisible by 3 since the sum of its digits is 4+3+2+1 = 10. It is 1 more than a multiple of 3. So the next multiple of 3 will be 4323. Hence could be 4323. But there are some other multiples of 3 which could be the value of . After 4323, 4326 and 4329 could also be the values of since they are multiples of 3 too. We know this because if A is a multiple of 3, A+3, A+6, A+9, A-3, A-6 etc are also multiples of 3. So since 4323 is a multiple of 3, 4326 and 4329 will also be multiples of 3. We did not get a unique value for so statement 1 alone is not sufficient.
Now let’s go on to statement 2.
This tells us that must be a multiple of 7. In 10 consecutive numbers, there will be either one multiple of 7 or two multiples of 7. If there is only one multiple of 7 in the range 4322 to 4331, statement 2 alone will be sufficient to give us the value of . If there are two multiples of 7 in this range, then statement 2 alone will not be sufficient.
Recall that from statement 1, we already know that will take one of three values: 4323, 4326 or 4329.
Let’s check for 4326 because it is in the middle. If 4326 is divisible by 7, there will be no other multiple of 7 in the range 4322 to 4331 because the closest multiples of 7 to 4326 will be 4326 – 7 and 4326 + 7. When we divide 4326 by 7, we find that it is divisible. This means that statement 2 gives us a single value of . Hence statement 2 alone is sufficient.
Hypothetically, what if we had found that 4326 is not divisible by 7? Then we would have known that either 4323 or 4329 must be a multiple of 7. In both cases, statement 2 would have given us 2 multiples of 7 because both 4330 (7 more than 4323) and 4322 (7 less than 4329) are in the possible range. Then we would have known that the answer will be (C) i.e. we will need both statements to answer the question since the possible values from the two statements will have only one overlap in either case.
Note that what we gleaned from statement 1 helped us quickly examine statement 2 and get to the answer right away. But this is an advanced technique and you should use it only if you understand it very well. Else, it is best to stick to completely ignoring one statement while working on the other.